La interpretación física de la condición de energía débil (WEC)

Estoy confundido por una declaración en "Spacetime and geometric" de Sean Carroll.

En la página 174-175, hace la siguiente declaración sobre la interpretación física del WEC en el caso de un fluido perfecto:

"Debido a que la presión es isotrópica, T m v t m t v será no negativo para todos los vectores temporales t m si ambos T m v tu m tu v 0 y T m v yo m yo v 0 para algún vector nulo yo m ".

T m v es el tensor de momento de energía y tu m es la velocidad cuatro. Para un fluido perfecto, las dos últimas desigualdades se reducen respectivamente a ρ 0 y ρ + pag 0 .

Entiendo que ambas desigualdades son condiciones necesarias para el WEC ( T m v t m t v 0 ) para sostener, pero obviamente no son condiciones suficientes .

Son condiciones necesarias como tu m es un vector temporal y, por continuidad, la desigualdad que implica yo m debe aguantar también. Sin embargo, se tratan como condiciones suficientes, es decir, el WEC se utiliza como sinónimo de " ρ 0 y ρ + pag 0 ".

En una métrica que no difiere demasiado de una métrica Minkowskiana (límite de campo débil), podemos descomponer cualquier vector temporal t m como una suma de un vector nulo bien elegido yo m y un múltiplo de tu m o

t m = yo m + λ tu m
con λ un número real Esto conduce directamente a
T m v t m t v = ρ ( λ tu m yo m ) 2 + pag ( tu m yo m ) 2
. Esta última ecuación implica que en el límite de campo débil, el WEC es equivalente a ρ 0 y pag 0 . ¿Alguien puede ayudarme a entender dónde está el error que estoy cometiendo y por qué las condiciones son condiciones suficientes?

Respuestas (2)

Ya has notado que son condiciones necesarias. Que sean suficientes se deduce precisamente porque la presión es isotrópica. Para ver esto, considere un marco local de Lorentz (ortonormal) (también conocido como tétrada). Cualquier vector nulo se puede escribir como

i = norte ( d 0 i + d 1 i ) ,
ya que no hemos especificado direcciones espaciales (en realidad, el 1 es solo un índice espacial elegido arbitrariamente), mientras que
tu i = d 0 i .
El tensor esfuerzo-energía es T i j = d i a gramo ( ρ , pag , pag , pag ) . Entonces
T i j tu i tu j = T 00 = ρ 0
y
T i j i j = norte 2 ( T 00 + T 11 ) = norte 2 ( ρ + pag ) 0
es equivalente a eso ρ + pag 0 . Finalmente, se puede descomponer un vector temporal arbitrario, como usted dice
t i = i + λ tu i ,
con λ 2 + 2 norte λ > 0 (igual a 1 si se normaliza), ya que en caso contrario t i sería nulo o espacial. De este modo
T i j t i t j = norte 2 ( ρ + pag ) + ( λ 2 + 2 norte λ ) ρ 0
por los resultados anteriores. Lo que significa que todos los observadores observan que la densidad de energía no es negativa.

Evaluar la desigualdad WEC en el marco local de Lorentz simplifica mucho las cosas y permite generalizar la afirmación sobre la descomposición del vector temporal. Sin embargo, evaluar el lado izquierdo de la última línea me lleva a
T m v t m t v = ρ ( norte + λ ) 2 + norte 2 pag
que es diferente de su resultado. Creo que mi error estuvo en la interpretación de esta última desigualdad. Tiene que ser válido para todos los vectores temporales, es decir, para cualquier valor de norte y λ , lo que me lleva a la conclusión de que ρ 0 y ρ + pag 0 .
@Jac ¡Ah, sí! Lo siento, estoy acostumbrado a la descomposición en vectores espaciales y temporales. ¡Buen lugar! De todos modos, la última desigualdad debería ser equivalente a los resultados anteriores (y por lo tanto necesariamente cierto dados los resultados anteriores). Corregí la respuesta para dar cuenta de esto.
De hecho, utilizando el hecho de que t m es temporal permite probar que las condiciones para ρ y ρ + pag no sólo son condiciones necesarias sino también suficientes. Todo está claro para mí ahora. Gracias.

Para futuros lectores de este hilo, aquí hay una descripción de cómo mostrar que la descomposición t m = yo m + λ tu m , que se usa en la respuesta aceptada, siempre existe (localmente).

Denotemos la norma del vector temporal t m por

T = t m t m < 0.

Deseamos encontrar un vector nulo yo m y un numero λ tal que t m = yo m + λ tu m . Entonces tendremos

yo m yo m = ( t m λ tu m ) ( t m λ tu m ) = t m t m 2 λ tu m t m + λ 2 tu m tu m .

Suponer t 0 = S en el marco de reposo de la fuente fluida perfecta del tensor de energía-momento, es decir, el marco en el que tu m = ( 1 , 0 , 0 , 0 ) m . Entonces

yo m yo m = T + 2 λ S λ 2 .

Para yo m para ser nulo necesitamos yo m yo m = 0 , y por lo tanto

λ 2 2 λ S T = 0.

Esta ecuación se puede resolver para λ siempre que S 2 + T 0 . Pero, usando coordenadas inerciales locales para evaluar T , encontramos eso

T = t m t m = ( t 0 ) 2 + i = 1 3 ( t i ) 2 S 2 ,

por lo que siempre es posible resolver para λ .